PT17.S3.Q24 - ditrama is a federation made up of three

Omed_OvOOmed_OvO Alum Member
edited April 2016 in General 158 karma
Any help would be great! Here's a link to the explanation page!

https://7sage.com/lsat_explanations/lsat-17-section-3-question-24

Thank you guys! :)

Comments

  • Cant Get RightCant Get Right Yearly + Live Member Sage 🍌 7Sage Tutor
    27823 karma
    Yeah, this is something they do pretty frequently, and this is a pretty hard one if you don’t have a really intuitive understanding of numbers and percentages.

    So let’s say it’s some years back and I own Joe’s Ebook Readers. It’s a small niche market, we custom make “computer tablets” on which people can read “digital books." Not a lot of interest yet, we’re the only ones who make them, but things are going well, we sold 1,000 last year.

    See where this is going?

    So the next year, Amazon releases the Kindle and they sell 15,000,000. My sales go from 1,000 to 1,200.

    So the population of Ditrama equates to the total number of all readers sold. Last year, Joe’s accounted for 100% of all sales. This year, Joe’s sold an additional 200 readers, but that now accounts for only 0.00008% of the population. The population increased from 1,000 to 15,001,200. So while my sales increased, my percentage decreased.
  • Omed_OvOOmed_OvO Alum Member
    158 karma
    @"Cant Get Right" thanks so much! Really helpful to see and understand.
  • Cant Get RightCant Get Right Yearly + Live Member Sage 🍌 7Sage Tutor
    27823 karma
    Yeah, the LSAT won’t ever provide you with anything so drastic, but this relationship between numbers and percentages always holds. Anytime you see a stim that’s talking about percentages or market share, you can almost guarantee the question is going to hinge on something like this. Using extremes is a good way to clearly expose what’s going on. So any time they tell me the population increases, I begin by assuming it increased by 15,000,000 or whatever large number happens to pop into my head. Then I look at how that might be distributed and it’s a lot easier to burn through the answer choices that way.
  • calftempcalftemp Free Trial Member
    edited May 2017 121 karma

    Hi @"Cant Get Right" + other 7sagers,

    Bumping this thread because I reviewed this question recently, and still find the answer choices pretty tricky. I'd appreciate any help. During BR, I drew out hypothetical scenarios with numbers and did the math, and I can see why TCR is what it is. That being said, I don't think I could ever have gotten there in ~ 1.5 minutes.

    When I read the stimulus, I almost immediately recognized that the increase in absolute number, but decrease in % meant that the denominator (total population of Ditrama) grew more/faster than the population of K. Thus, the populations of M + G must've grown more than K!

    However, I really hesitated to pick an answer that relied on % (eg: C & E), thinking that we don't know anything about the relative sizes of K, M, & G - so we can't really say anything about relative % growth for the different regions. Down to answer choices only about numbers, I ended up picking D, because it was about the numerical CHANGE, not the absolute total number.

    Any tips/advice on how to solve this question in 1:25 or so, without doing math during the test? Correct me if I'm wrong, but I feel like LSAC doesn't want us to do actual math during the test...

    Thanks in advance!

  • dcdcdcdcdcdcdcdcdcdc Alum Member
    382 karma

    @calftemp

    I'll give this shot.

    I don't think you need to be so wary of absolute numbers vs. percentages for this question. The use of "share" and "percentage" are interchangeable in the stimulus and the only mention of absolute numbers is the increase in population in Korva.

    I approached this almost like an RRE type question beacuse we might expect that, given the increase in population, Korva's budget certainly should not decrease. The absolute number that you mention for Korva's population increase can also be expressed as a percentage increase; that is just a different way of viewing the same data point.

    In this case, we know that Korva's population must now represent a smaller percentage of the total population of Ditrama. Why? We are told in the stimulus that the budget is apportioned to be representative of the share of the population that each region contains, so therefore if Korva's budget is reduced, it must represent a smaller proportion of the overall nation's population.

    However, we also know the population of Korva increased! So if we are still going to reduce it's budget, it must be that the other members, in some combination, comprise a larger total of the percentage. Because of the reapportioning of the budget, we can tell that one of or possibly both other regions grew their population by more than Korva.

    Let's break that down more: if the other regions only grew by the same percentage as Korva, then the budget would not be reapportioned. If they grew by less than Korva, then Korva would have its budget increased, as opposed to decreased.

  • Mellow_ZMellow_Z Alum Member
    edited May 2017 1997 karma

    @dcdcdcdcdc said:
    @calftemp

    I'll give this shot.

    I don't think you need to be so wary of absolute numbers vs. percentages for this question. The use of "share" and "percentage" are interchangeable in the stimulus and the only mention of absolute numbers is the increase in population in Korva.

    I approached this almost like an RRE type question beacuse we might expect that, given the increase in population, Korva's budget certainly should not decrease. The absolute number that you mention for Korva's population increase can also be expressed as a percentage increase; that is just a different way of viewing the same data point.

    In this case, we know that Korva's population must now represent a smaller percentage of the total population of Ditrama. Why? We are told in the stimulus that the budget is apportioned to be representative of the share of the population that each region contains, so therefore if Korva's budget is reduced, it must represent a smaller proportion of the overall nation's population.

    However, we also know the population of Korva increased! So if we are still going to reduce it's budget, it must be that the other members, in some combination, comprise a larger total of the percentage. Because of the reapportioning of the budget, we can tell that one of or possibly both other regions grew their population by more than Korva.

    Let's break that down more: if the other regions only grew by the same percentage as Korva, then the budget would not be reapportioned. If they grew by less than Korva, then Korva would have its budget increased, as opposed to decreased.

    Good explanation. This type of question just takes time. The more comfortable you are with numbers and percents, the easier it will be to solve for you. Here's a quick example I drew up to show that both of M and G don't have to increase, to cause K's portion to decrease. C was the answer choice that originally threw me off, and is the one that most people mistakenly choose, so hopefully it helps someone.

    http://i.imgur.com/fGabSMi.png

    I think it's also worth emphasizing that K doesn't have to increase at all. So this scenario would throw D out as well as C. Just because K's population increased, doesn't mean that both M and G increased as well. So if K increased by 10%, and lets say M increases by 1000%, but G has a 0% increase. K's portion will decrease, M's will increase, and G's will decrease. Since not all 3 increased, the verbiage of C and D excludes them from being possibly correct.

    http://i.imgur.com/pMlEpNz.png

Sign In or Register to comment.